LSAT and Law School Admissions Forum

Get expert LSAT preparation and law school admissions advice from PowerScore Test Preparation.

 Administrator
PowerScore Staff
  • PowerScore Staff
  • Posts: 8950
  • Joined: Feb 02, 2011
|
#40626
Complete Question Explanation
(The complete setup for this game can be found here: lsat/viewtopic.php?t=14136)

The correct answer choice is (B)

Although Rule Substitution questions can be tricky, this is one of the easiest Rule Substitution questions to appear on an LSAT.

The question stem asks you to replace the second rule with a condition that will have an identical effect. The second rule eliminates G from being the fourth school to which juices are delivered. This rule, in combination with the inference that H or I must be the first school to which juices are delivered, limits G to being the second or third school to which juices are delivered. Thus, we need to preserve that result within the juices row without adding any new restrictions. Answer choice (B) states that result exactly, and thus answer choice (B) is the correct answer. However, let’s look at each of the incorrect answers as well.

Answer choice (A): This answer choice does not restrict G in the same way as the original rule, and in addition further restricts F in a way that it was not previously limited.

Answer choice (C): This answer choice does not restrict G in the same way as the original rule, and in addition further restricts H in a way that it was not previously limited.

Answer choice (D): This answer choice does not restrict G in the same way as the original rule, and in addition forces I to be the first school to which juices are delivered and the fourth school to which snacks are delivered. This is an additional restriction that did not exist originally.

Answer choice (E): This answer choice appears promising at first, because it eliminates G from being the fourth school to which juices are delivered. However, it also eliminates F from being the fourth school to which juices are delivered, and that is not a restriction that existed with the original rule.
 CEF
  • Posts: 9
  • Joined: Sep 03, 2013
|
#11717
I was wondering what the best approach is to these types of questions? They are the ones I typically miss because I'm not sure what the best method of attack is.

Thank you!
User avatar
 Dave Killoran
PowerScore Staff
  • PowerScore Staff
  • Posts: 5972
  • Joined: Mar 25, 2011
|
#11720
Hi CEF,

That's a Rule Substitution question. I'm going to borrow from a response I made to another student asking about those:

From http://forum.powerscore.com/lsat/viewto ... 506#p11506
  • Those are Rule Substitution questions. Here is a list of the ten that have ever appeared on the LSAT: http://www.powerscore.com/gamesbible/lo ... /index.cfm.

    An expansive breakdown of how to approach those questions is a bit beyond the scope of the forum, but the newest version of LSAT Logic Games Bible, in pages 450 through 460, contains a detailed analysis of Rule Substitution questions, the various answers that appear, and how to handle them.

    The Online Student Centers for our Full-length and Live Online courses contains a breakdown of those questions in the Lesson 11 Homework supplement sections.
This specific question interestingly used a concept I explained in the new LGB, which at the time hadn't yet been used in one of these questions. So we predicted the future a bit :-D

Thanks!
User avatar
 lemonade42
  • Posts: 95
  • Joined: Feb 23, 2024
|
#106073
Hello,

For explaining why (D) is wrong, why does it say "and in addition forces I to be the first school to which juices are delivered and the fourth school to which snacks are delivered. This is an additional restriction that did not exist originally". I thought I was allowed to be at the 1st school for juices and the 4th school for snacks before? I thought it was F that couldn't be at 1st school of juices and 4th school of snacks before.
User avatar
 Dana D
PowerScore Staff
  • PowerScore Staff
  • Posts: 385
  • Joined: Feb 06, 2024
|
#106090
Hey Lemonade,

You're exactly right, "I" can be the first school to which juices are delivered, and "F" cannot be, meaning if this rule was added to the game, the only option for the first juice delivery would be I, otherwise we'd be breaking other rules. This is a restriction that didn't exist before - we still couldn't ever have F be the first getting juice, but under the original rules we did have the option for H to get the first juice delivery without breaking any other rules. If we added this rule, we'd be making it so H can't be first either, which is part of the reason answer choice (D) is wrong.

Hope that helps!

Get the most out of your LSAT Prep Plus subscription.

Analyze and track your performance with our Testing and Analytics Package.